PT 36 - Dec 2001 Sec.1 Q18 Forum

Prepare for the LSAT or discuss it with others in this forum.
Post Reply
User avatar
Ragged

Silver
Posts: 1496
Joined: Wed Oct 21, 2009 12:39 pm

PT 36 - Dec 2001 Sec.1 Q18

Post by Ragged » Sun May 09, 2010 11:28 pm

Please explain to me what the correct answer is and why.

Cambridge LSAT

Bronze
Posts: 257
Joined: Mon Aug 24, 2009 3:26 pm

Re: PT 36 - Dec 2001 Sec.1 Q18

Post by Cambridge LSAT » Mon May 10, 2010 12:29 am

Conclusion: moderate exercise lowers the risk of arterial blockage due to blood clots

P1: lowers blood cholesterol → lowers the risk of hardening arteries → lowers the risk of arterial blockage due to blood clots
P2: data are correct → moderate exercise lowers blood cholesterol

If the sufficient condition of P2 is assumed, we can plug moderate exercise into the front end of the P1 chain:
moderate exercise → lowers blood cholesterol → lowers the risk of hardening arteries → lowers the risk of arterial blockage due to blood clots

Thus, choice D is correct.

User avatar
Ragged

Silver
Posts: 1496
Joined: Wed Oct 21, 2009 12:39 pm

Re: PT 36 - Dec 2001 Sec.1 Q18

Post by Ragged » Mon May 10, 2010 2:42 am

Thank you Cambridge. I'm an idiot. I had all the relationships worked out but I failed to realize that the first sentence, not the last, was the conclusion. Go me.

Post Reply

Return to “LSAT Prep and Discussion Forum”